brandon06223
brandon06223
02-04-2019
Mathematics
contestada
Please i need help i don’t understand
Respuesta :
lilwolfie65
lilwolfie65
02-04-2019
W= 2z+3
BecAuse the 2 is on the z axis and I got +3 because the dots are on 5 and 8 and 5+3=8
Answer Link
VER TODAS LAS RESPUESTAS ( 19+ )
Otras preguntas
A 21-year-old woman presents to the emergency department with fevers, headache, neck stiffness, and mild confusion over the past several days. Her temperature i
Which sentence best describes the French sale of the Louisiana Purchase to the United States? Select one a. Napoleon was pleased with the money earned and that
Ceasar opens a bank account with an initial deposit of $1800.00 with an annual interest rate of 3.2%. What is the balance on the account after one year?
look at the image below for the question !!!!!!
How have people been lately good? bad? something else?
khái quát về lịch sử hình thành và phát triển của một cơ quan nhà nước
How much power does it take to lift a box with a weight of 50N to put it on a shelf that is 10m high in 10 sec?
What is the fallopian tube fimbriae made of? Pls I need help asap
The total number of restaurant-purchased meals that the average person will eat in a restaurant, in a car, or at home in a year is 193 . The total number of
50 points pls help. question down below explaination is option but with if u want brainlist u know